Đến nội dung

OiDzOiOi nội dung

Có 105 mục bởi OiDzOiOi (Tìm giới hạn từ 22-05-2020)



Sắp theo                Sắp xếp  

#622649 Đề thi hsg toán 8 Nam Định

Đã gửi bởi OiDzOiOi on 25-03-2016 - 22:18 trong Tài liệu - Đề thi

3.    $=(2y-x)^{2}+3(x-\frac{1}{2})^{2}-\frac{3}{4}$




#623530 Đề thi hsg toán 8 Nam Định

Đã gửi bởi OiDzOiOi on 29-03-2016 - 22:12 trong Tài liệu - Đề thi

$A=n.4^{n}+3^{n}=n.(7-3)^{n}+3^{n}=n.B(7)+n.(-3)^{n}+3^{n}$. Vì A $\vdots$ 7 nên $n.(-3)^{n}+3^{n}\vdots 7$

Với n chẵn $A=3^{n}(n+1) \vdots 7$$ \Rightarrow(n+1)\vdots7$ hay $n+1=7(2k+1)$ (vì n+1 lẻ ) $\Rightarrow n=14k+6$ ( k nguyên) 

Với n lẻ $A=3^{n}(1-n)$ tương tự $\Rightarrow 1-n=7.2k=14k $$\Leftrightarrow n=1-14k  $




#622631 Đề thi hsg toán 8 Nam Định

Đã gửi bởi OiDzOiOi on 25-03-2016 - 22:03 trong Tài liệu - Đề thi

$\frac{1}{m+n-x}=\frac{1}{m}+\frac{1}{n}-\frac{1}{x}\Leftrightarrow \frac{1}{m+n-x}-\frac{1}{m}=\frac{x-n}{xn}\Leftrightarrow \frac{x-n}{x+n-m}=\frac{x-n}{xn}\Leftrightarrow \begin{bmatrix} x=n & & \\m( m+n-x)=xn& & \end{bmatrix}$

Với x=n thì mọi m đều thỏa mãn

Với m(m+n-x)=xn tương đương (m+n)(m-x)=0




#623517 Đề thi hsg toán 8 Nam Định

Đã gửi bởi OiDzOiOi on 29-03-2016 - 21:50 trong Tài liệu - Đề thi

3.       $\sum \frac{1}{2x+y+z}=\sum \frac{1}{4}.\frac{4}{(x+y)+(x+z)}\leq \frac{1}{4}.\sum (\frac{1}{x+y}+\frac{1}{x+z})= \frac{1}{2}\sum \frac{1}{x+y}=\frac{1}{8}.\sum \frac{4}{x+y}\leq \frac{1}{8}.\sum (\frac{1}{x}+\frac{1}{y})=\frac{1}{4}.\sum \frac{1}{x}=1$




#601805 Đề thi giải toán trên máy tính cầm tay thành phố BH-tỉnh ĐN năm học 2015-216

Đã gửi bởi OiDzOiOi on 05-12-2015 - 21:43 trong Giải toán bằng máy tính bỏ túi

Bạn biết làm bài 4 với bài 7 ko, chỉ mik làm với

bài 7   đặt $x+\frac{1}{x}=a$

      $y+\frac{1}{y}=b$

      $\left\{\begin{matrix} a+b & & =4,9239\\ a^{2}+b^{2}&&=12,4648 \end{matrix}\right.$

      $\Rightarrow x^{3}+y^{3}+\frac{1}{x^{3}}+\frac{1}{y^{3}}=(a+b)(a^{2}-((a+b)^{2}-(a^{2}+b^{2}))+b^{2})-(a+b)$

Thay vào là ra




#593134 Bất Phương Trình

Đã gửi bởi OiDzOiOi on 10-10-2015 - 21:49 trong Phương trình - hệ phương trình - bất phương trình

là sao? bạn giải kỹ hơn giùm mình đi




#593128 Bất Phương Trình

Đã gửi bởi OiDzOiOi on 10-10-2015 - 21:43 trong Phương trình - hệ phương trình - bất phương trình

Cho các số thực dương a, b, c. Chứng minh rằng

 

$\dpi{200} \frac{a^{2}}{b+c}+\frac{b^{2}}{a+c}+\frac{c^{2}}{a+b}\geq \frac{c^{2}}{b+c}+\frac{a^{2}}{c+a}+\frac{b^{2}}{a+b}$




#593129 Bất Phương Trình

Đã gửi bởi OiDzOiOi on 10-10-2015 - 21:44 trong Phương trình - hệ phương trình - bất phương trình

Cho các số thực dương a, b, c. Chứng minh rằng

 

$\ \frac{a^{2}}{b+c}+\frac{b^{2}}{a+c}+\frac{c^{2}}{a+b}\geq \frac{c^{2}}{b+c}+\frac{a^{2}}{c+a}+\frac{b^{2}}{a+b}$




#596887 Bài 1. Chứng minh rằng số đường chéo của một đa giác lồi n cạnh là $...

Đã gửi bởi OiDzOiOi on 04-11-2015 - 22:10 trong Số học

đa giác n cạnh => có n đỉnh... 
mỗi đỉnh of đa giác có thể nối với (n-3) đỉnh khác để tạo ra (n-3) đường chéo....(trừ đỉnh ta đang xét và 2 đỉnh gần nhất....(vì nối tạo ra cạnh)) 
ta có n đỉnh => sẽ có n.(n-3) đường chéo.. 
nhưng 1 đường chéo sẽ đc nối bởi 2 đỉnh => số đg chéo sẽ đc nhân đôi = n.(n+3) 
=> số đường chéo thực = n.(n-3)\2




#593127 Ai Giúp Mình Giải Bài Này Với !

Đã gửi bởi OiDzOiOi on 10-10-2015 - 21:37 trong Phương trình - hệ phương trình - bất phương trình

Giải hệ phương trình

 

$\ \left\{\begin{matrix} x+y+z=6 & \\ \frac{xy+xz+yz}{\sqrt{x}+\sqrt{y}+\sqrt{z}}=2\sqrt{2} & \end{matrix}\right.$




#597883 2q+q2​=r

Đã gửi bởi OiDzOiOi on 11-11-2015 - 21:28 trong Số học

Với q>3 ; Tìm 2 số nguyên tố q và r biết 2q+q2=r




#597896 2q+q2​=r

Đã gửi bởi OiDzOiOi on 11-11-2015 - 21:50 trong Số học

Vì q>3 nên 3 không chia hết cho q, do đó $k\vdots q$, vậy (k,q)=1 là sai!

(k;q)=1 thì 3 chia hết q suy ra q=3 thì mới kết luận không tồn tại q mà bạn




#597886 2q+q2​=r

Đã gửi bởi OiDzOiOi on 11-11-2015 - 21:34 trong Số học

Hướng giải 

$2^{q}+q^{2}\equiv 2$ ( mod 3 ) $\Rightarrow r-2=3k$

Mặt khác: theo định lý nhỏ Fermat: $2^{q}-2 \vdots q \Rightarrow r-2\vdots q$

Do đó $3k\vdots q$ đến đây làm sao để chứng minh (k;q)=1 vậy chỉ mình với




#598015 2q+q2​=r

Đã gửi bởi OiDzOiOi on 12-11-2015 - 20:08 trong Số học

Cần j đến Phéc-ma nhỉ , theo mình làm như sau nhé 
Vì với n>3 và $q^2+2^q$ là snt nên q lẻ suy ra $2^q \equiv 2(mod3)$
Ta lại có q ko chia hết cho 3 thì suy ra $q^2$ \equiv 1 (mod3) 
=> $q^2+2^q$ chia hết cho 3 ( vô lí)
Vậy không có q,r thỏa mãn đề ra :v



Cách này biết lâu rồi. Chỉ là tìm cách mới thôi



#619450 $T=\frac{x^{2}+3y^{2}}{2xy^...

Đã gửi bởi OiDzOiOi on 09-03-2016 - 23:31 trong Bất đẳng thức và cực trị

 x,y $\in$ IR+,  x+y=2

 

Find Min :

 

$T=\frac{x^{2}+3y^{2}}{2xy^{2}-x^{2}y^{3}}$




#623104 $s_{1}=2$, $s_{2}=2+5$,...

Đã gửi bởi OiDzOiOi on 27-03-2016 - 22:55 trong Đại số

có thể làm như này 

$y=\frac{p+1}{2} \Rightarrow S_{n+1} =(\frac{p+1}{2})^2$

Mà do $n \geq 2 \Rightarrow S_{n+1}=(2+3+5+...+p) \leq (1+3+5+...+p)+2-9-1=1^2-0^2+2^2-1^2+3^2-2^2+...+(\frac{p+1}{2})^2-(\frac{p-1}{2})^2-8 < (\frac{p+1}{2})^2$




#623098 $s_{1}=2$, $s_{2}=2+5$,...

Đã gửi bởi OiDzOiOi on 27-03-2016 - 22:42 trong Đại số

Giả sử ngược lại gọi hai tổng đó là $S_n=x^2,S_{n+1}=y^2$  (quy ước $x,y$ nguyên dương)
Khi đó $S_{n+1}-S_n=p$ (quy ước $p$ là số nguyên tố) 
Hay $(y-x)(y+x)=p$ suy ra $y-x=1,y+x=p$ từ đó suy ra $y=\frac{p+1}{2},x=\frac{p-1}{2}$ 
Suy ra $\sqrt{S_n}+\sqrt{S_{n+1}}=\sqrt{p^2}$ . Điều này xảy ra chỉ khi $S_n=0$ (vô lí)
Vậy ta có đpcm

sao lại xảy ra chỉ khi $S_{n}=0$ thôi hả bạn




#623081 $s_{1}=2$, $s_{2}=2+5$,...

Đã gửi bởi OiDzOiOi on 27-03-2016 - 22:04 trong Đại số

Với mỗi số nguyên dương n , ký hiệu $S_{n}$ là tổng n số nguyên tố đầu tiên

$S_{1}=2$ ; $S_{2}=2+3$  ;  $S_{3}=2+3+5$, ...

Chứng minh rằng trong dãy số $S_{1}$ , $S_{2}$, $S_{3}$, ... không tồn tại hai số hạng liên tiếp đều là số chính phương  




#594942 $d\leqslant \sqrt{a+b}$

Đã gửi bởi OiDzOiOi on 23-10-2015 - 12:07 trong Bất đẳng thức và cực trị

Cho $a;b;c \in N^{*}$ sao cho $\frac{a+1}{a}+\frac{b+1}{b}\in Z$

Gọi d là ước chung của a và b.

Chứng minh $d\leqslant \sqrt{a+b}$




#621494 $a^{2}+b^{2}=\begin{bmatrix} a,b...

Đã gửi bởi OiDzOiOi on 20-03-2016 - 21:13 trong Số học

$a,b\in N^{*}$        Fnd ::        $a^{2}+b^{2}=\begin{bmatrix} a,b \end{bmatrix}+7(a,b)$

(  $[a,b]=BCNN(a,b)$ ;    $(a,b)=UCLN(a,b))$ )




#610853 $6x^2+5y^2=74 $

Đã gửi bởi OiDzOiOi on 24-01-2016 - 22:10 trong Số học

Giải các phương trình nghiệm nguyên sau

1.        6x2+5y2=74 

2        .x2+xy+y2 =x+8y

3        .1+x+x2+x3=y3

4       .1+x+x2+x3+x4=y2

5.       (x-2)4-x4=y3

6.         $\sqrt{x}+\sqrt{y-1}+\sqrt{z-1}=\frac{1}{2}(x+y+z)$ 

7.     x2+y2 +z2=x2y2




#610861 $6x^2+5y^2=74 $

Đã gửi bởi OiDzOiOi on 24-01-2016 - 22:23 trong Số học

3) $ (x+1)^3 \ge y^3=x^3+x^2+x+1>(x-1)^3 \Rightarrow x=0,y=1$ 

x có thể âm sao so sánh được




#594762 $1+x^{4}=2(1+x)^{4}$

Đã gửi bởi OiDzOiOi on 21-10-2015 - 22:00 trong Phương trình, hệ phương trình và bất phương trình

Giải phương trình  

 

$1+x^{4}=2(1+x)^{4}$




#595967 $\sum \frac{b+1}{a+b+1} \geq 2$

Đã gửi bởi OiDzOiOi on 29-10-2015 - 18:43 trong Bất đẳng thức và cực trị

Ặc.Sai rồi




#595966 $\sum \frac{b+1}{a+b+1} \geq 2$

Đã gửi bởi OiDzOiOi on 29-10-2015 - 18:41 trong Bất đẳng thức và cực trị

Bài 1:

Ta chứng minh được $\frac{a}{b+1}+\frac{b}{c+1}+\frac{c}{a+1}\geq \frac{3}{2}$

Do đó $\sum \frac{b+1}{a+b+1}=\sum \frac{1}{\frac{a+b+1}{b+1}}\geq \frac{9}{\sum \frac{a}{b+1}+1}\geq 2$